Manager: I recommend that our company reconsider the decision to completely abandon our allegedly difficult–to–use c...

tselimovic on February 4, 2015

Clarification

Why must (E) be true?

Replies
Create a free account to read and take part in forum discussions.

Already have an account? log in

Naz on February 10, 2015

We know that the manager does not prefer the new software to the one that they are currently using. Why? He states that several other companies in their region have replaced the software that they currently use with the new software and even though their employees can all use the new software, unofficially many of them continue to use their former software as much as possible.

Well, remember that the term "many" is a type of "some." And we know that the definition of "some" in our LSAT world is "at least one." Thus, since, the manager is technically one of the employees in the company, he counts as one employee in the company who would not prefer the new software package to the software currently in use.

Therefore, answer choice (E): "Many of the employees in the manager's company would not prefer the new software package to the software currently in use," must be true.

Again, since "many" could be "at least one," we know this must be true because we know that at least the manager would not prefer the new software package to the software currently in use.

Hope that clears things up! Please let us know if you have any other questions.

dfolave on January 29, 2016

What's wrong with B?

joryjes on September 22, 2018

And then no answer? What is wrong with B? Can someone this question from two years ago please?

Mehran on September 24, 2018

@joryjes (B) is much too broad to pass the must be true test. It is a general statement about which is a more important consideration, flexibility/ease of use or familiarity.

Nothing in the stimulus would support such a broad claim. We only know this manager's opinion.

Hope that helps! Please let us know if you have any other questions.

ikarus on June 14, 2020

In @Naz's explanation, she stated that "many" is the same as "some". For clarification, in LSAT world "many" = "some" = "at least one", so "many" = " at least one"?